Difference between revisions of "2018 AMC 10B Problems/Problem 21"

(Solution)
(Solution)
Line 6: Line 6:
 
==Solution==
 
==Solution==
  
Answer: (B) 340
+
Prime factorizing <math>323</math> gives you <math>17 \cdot 19</math>. Looking at the answer choices, only <math>\fbox{(B) 340}</math> is divisible by <math>17</math>.
  
 
==See Also==
 
==See Also==

Revision as of 16:45, 16 February 2018

Problem

Mary chose an even $4$-digit number $n$. She wrote down all the divisors of $n$ in increasing order from left to right: $1,2,...,\dfrac{n}{2},n$. At some moment Mary wrote $323$ as a divisor of $n$. What is the smallest possible value of the next divisor written to the right of $323$.

$\textbf{(A) } 324 \qquad \textbf{(B) } 330 \qquad \textbf{(C) } 340 \qquad \textbf{(D) } 361 \qquad \textbf{(E) } 646$

Solution

Prime factorizing $323$ gives you $17 \cdot 19$. Looking at the answer choices, only $\fbox{(B) 340}$ is divisible by $17$.

See Also

2018 AMC 10B (ProblemsAnswer KeyResources)
Preceded by
Problem 20
Followed by
Problem 22
1 2 3 4 5 6 7 8 9 10 11 12 13 14 15 16 17 18 19 20 21 22 23 24 25
All AMC 10 Problems and Solutions
2018 AMC 12B (ProblemsAnswer KeyResources)
Preceded by
Problem 18
Followed by
Problem 20
1 2 3 4 5 6 7 8 9 10 11 12 13 14 15 16 17 18 19 20 21 22 23 24 25
All AMC 12 Problems and Solutions

The problems on this page are copyrighted by the Mathematical Association of America's American Mathematics Competitions. AMC logo.png